edited by
456 views
2 votes
2 votes

$Q8$ If $\alpha_{1},\alpha_{2},\alpha_{3}, \dots , \alpha_{n}$ be the roots of $x^{n}+1=0$, then $\left ( 1-\alpha_{1} \right )\left ( 1-\alpha_{2} \right ) \dots \left ( 1-\alpha_{n} \right )$ is equal to

  1. $1$
  2. $0$
  3. $n$
  4. $2$
edited by

1 Answer

1 votes
1 votes

$x^{n}+1=0$ ------------$>(1)$

$\text{Case1:}$ If $n$ is odd $\text{(let n=3)}$

$x^{3}+1=0$

We can write like this

$x^{3}+0\cdot x^{2}+0\cdot x +1=0$ -----------$>(2)$

$(1-\alpha_{1})(1-\alpha_{2})(1-\alpha_{3})=1-\alpha_{1}-\alpha_{2}+\alpha_{1}\alpha_{2}-\alpha_{3}+\alpha_{1}\alpha_{3}-\alpha_{1}\alpha_{2}\alpha_{3}+\alpha_{2}\alpha_{3}$

                                                    $=1-\alpha_{1}-\alpha_{2}-\alpha_{3}+\alpha_{1}\alpha_{2}+\alpha_{1}\alpha_{3}+\alpha_{2}\alpha_{3}-\alpha_{1}\alpha_{2}\alpha_{3}$

                                                   $=1-(\alpha_{1}+\alpha_{2}+\alpha_{3})+(\alpha_{1}\alpha_{2}+\alpha_{2}\alpha_{3}+\alpha_{1}\alpha_{3})-\alpha_{1}\alpha_{2}\alpha_{3}$ -----$>(3)$

                     $\text{From equation $(2)$}$

$$\alpha_{1}+\alpha_{2}+\alpha_{3}=\frac{-b}{a}=\frac{0}{1}=0$$

$$\alpha_{1}\alpha_{2}+\alpha_{2}\alpha_{3}+\alpha_{1}\alpha_{3}=\frac{c}{a}=\frac{0}{1}=0$$

$$\alpha_{1}\alpha_{2}\alpha_{3}=\frac{-d}{a}=\frac{-1}{1}=-1$$

Put these value into equation $(3)$ we get

$(1-\alpha_{1})(1-\alpha_{2})(1-\alpha_{3})=1-(0)+(0)-(-1)=1+1=2$ 

 

$\text{Case2:}$ If $n$ is even $\text{(let n=2)}$

$x^{2}+1=0$

We can write like this

$x^{2}+0\cdot x +1=0$ -----------$>(4)$

$(1-\alpha_{1})(1-\alpha_{2})=1-\alpha_{1}-\alpha_{2}+\alpha_{1}\alpha_{2}$

                                  $=1-(\alpha_{1}+\alpha_{2})+\alpha_{1}\alpha_{2}$ -------------$>(5)$

                            $\text{From equation $(4)$}$

$$\alpha_{1}+\alpha_{2}=\frac{-b}{a}=\frac{0}{1}=0$$

$$\alpha_{1}\alpha_{2}=\frac{c}{a}=\frac{1}{1}=1$$

Put these value into equation $(5)$ we get

$(1-\alpha_{1})(1-\alpha_{2})=1-(0)+1=2$

So$,$answer is $2$

Option $(D)$ is the correct choice$.$

edited by

Related questions

2 votes
2 votes
1 answer
1
Tesla! asked Apr 3, 2017
516 views
$Q10$ The equation $p\left ( x \right ) = \alpha$ where $p\left ( x \right ) = x^{4}+4x^{3}-2x^{2}-12x$ has four distinct real root if and only if$p\left ( -3 \right )<\...
1 votes
1 votes
1 answer
2
Tesla! asked Apr 3, 2017
375 views
If the equation $x^{4}+ax^{3}+bx^{2}+cx+1=0$ (where $a,b,c$ are real number) has no real roots and if at least one of the root is of modulus one, then$b=c$$a=c$$a=b$none ...
6 votes
6 votes
2 answers
3
Tesla! asked Apr 3, 2017
652 views
The equation $\frac{1}{3}+\frac{1}{2}s^{2}+\frac{1}{6}s^{3}=s$hasexactly three solution in $[0.1]$exactly one solution in $[0,1]$exactly two solution in $[0,1]$no solu...
5 votes
5 votes
2 answers
4
Tesla! asked Apr 3, 2017
1,814 views
The equation $x^{6}-5x^{4}+16x^{2}-72x+9=0$ hasexactly two distinct real rootsexactly three distinct real rootsexactly four distinct real rootssix different real roots